LSAT and Law School Admissions Forum

Get expert LSAT preparation and law school admissions advice from PowerScore Test Preparation.

 Administrator
PowerScore Staff
  • PowerScore Staff
  • Posts: 8917
  • Joined: Feb 02, 2011
|
#81055
Complete Question Explanation

Strengthen, Principle. The correct answer choice is (A).

Answer choice (A): This is the correct answer choice.

Answer choice (B):

Answer choice (C):

Answer choice (D):

Answer choice (E):

This explanation is still in progress. Please post any questions below!
 nihals23
  • Posts: 16
  • Joined: Oct 01, 2018
|
#59218
Option A and B say practically the same thing. Voters are unlikely to notice the change in the politician's stance i.e. they still think shes insincere. So is answer A right just because it uses the word viz 'insincere' used in the passage? Please do reply, I used up some precious time on the test on this.
 Malila Robinson
PowerScore Staff
  • PowerScore Staff
  • Posts: 296
  • Joined: Feb 01, 2018
|
#59445
Hi Nihals23,
Answer A and B seem to contradict each other. If Answer B were true and voters are unlikely to notice that a politician's stance has changed over time then they would not have a reason to feel that the politician was insincere (Answer A) due to a changed position (because again, they wouldn't have noticed the change).

The passage is stating that voters will consider the mayoral candidate to be insincere because they noticed that her position shifted, and they don't believe that the shift is because she learned more about the issue. That leads to Answer A as the correct choice.

Hope that helps!
-Malila

Get the most out of your LSAT Prep Plus subscription.

Analyze and track your performance with our Testing and Analytics Package.